What is Bracket: Definition and 83 Discussions

A bracket is either of two tall fore- or back-facing punctuation marks commonly used to isolate a segment of text or data from its surroundings. Typically deployed in symmetric pairs, an individual bracket may be identified as a left or right bracket or, alternatively, an opening bracket or closing bracket, respectively, depending on the directionality of the context.
Specific forms of the mark include rounded brackets (also called parentheses), square brackets, curly brackets (also called braces), and angle brackets (also called chevrons), as well as various less common pairs of symbols.
As well as signifying the overall class of punctuation, the word bracket is commonly used to refer to a specific form of bracket, which varies from region to region. In most English-speaking countries, an unqualified 'bracket' refers to the round bracket; in the United States, the square bracket.

View More On Wikipedia.org
  1. A

    Integration Problem involving absolute bracket, sine,cosine

    Homework Statement Evaluate ∫(1)to(-1) l sin(x) l dx Homework Equations The Attempt at a Solution If I just leave it like this, is it wrong? 1. ∫(1)to(-1) l sin(x) l dx = l -cos (x)l 2. l -cos (1) l — l -cos (-1)l = 0 - 0 = 0
  2. maverick_starstrider

    Poisson Bracket to Commutator, What Does it REALLY Mean?

    Let me just head off the first waves of posts this thread will likely get. I am very fluent in quantum mechanics. I am completely aware of the behaviour of a commutator structure: simultaneous eigenbasis, etc. I understand how commutators model the structure that quantum mechanics has. My...
  3. A

    Angular momentum in bracket notation

    Hello, why is <j' m' | j m>=0 if j \not= j' \; \text{or} \; m \not= m' j and m are describing states. I don't understand why an 'empty bracket' behaves like that.
  4. WannabeNewton

    Show Lie Bracket of X & Y is Linear Comb. of Commuting Vector Fields

    Homework Statement Show that if the vector fields X and Y are linear combinations (not necessarily with constant coefficients) of m vector fields that all commute with one another, then the lie bracket of X and Y is a linear combination of the same m vector fields. The Attempt at a Solution...
  5. A

    Using Dirac Bracket: Books & References for Examples

    I have searched in web and go through some papers. But the use of Dirac Bracket in constraint still unclear to me. It would be better if I have some examples. Can anyone please help me by suggesting books/references where I can find details about using Dirac Bracket?
  6. S

    Strengthing metal engine harness bracket to support intake pipe

    hello! My first post here. I am by no means a mechanical engineer as I studied computer science in college. My question has to do with adding what is known as a cold air intake onto my vehicle. The design of the intake is such that the pipe fits at the throttle body via a silicone/rubber...
  7. D

    Bracket Predictions: Picking an Underdog to Win it All!

    How is everyone's bracket looking. Right now I'm in the 85th percentile for espn and I'm happy with that. Final Four: Syracuse over Duke Kansas over Wisconsin Championship: Syracuse over Kansas I don't actually think Syracuse will win but to win pools with tons of people it is smart...
  8. R

    Calculating Force Transmitted by Rubber Mountings to Failed Bracket

    I'm having difficulties in finding an answer to the issue show in the attached pic. Fracture surface on the failed bracket! I have the properties of the mild steel, the properties of the rubber mountings, and the number of the cycles per minute. What i can't figure out is the load or the force...
  9. 1

    How can I calculate the stress on a bracket during impact with a moving cart?

    Hi all, I am trying to calculate a value for stress in a right angled bracket. this would be simple for me if the load was a static load, it would cause the bracket to bend around a point etc so i can get moments to find its bending stress. however this bracket will be stopping a moving...
  10. A

    Nijenhuis bracket - am I crazy?

    So, I am learning about tangent-valued differential forms, operations on forms and Frolicher-Nijenhuis bracket, and I am writing a blog about what I have learned. In two monographs I have spotted a formula that - as far as can see - has a missing factor. At least this is how I understand what...
  11. D

    How Effective is Blocking Side Gaps in Vent Filter Systems?

    I am designing these brackets on a ventilation system to hold filters that my company stocks regularly. The vent is about 49.5"x49.5" and the filters are about 23.5"x23.5". Placing 4 filters in this square vent leaves about 1.25" on each side. I have designed the brackets so that they block the...
  12. T

    Equilibrium of a T-Shaped Bracket: Determining Reactions at A and C

    Homework Statement A T-shaped bracket supports a 150-N load as shown. Determine the reactions at A and C when (a) \alpha=90o, (b) \alpha=45o ans: (a)A=150N going down, C=167.7N,63.4degrees (b)A= 194.5N going down; C=253N, 77.9degrees base on the book...
  13. K

    Will this bracket support a swingset?

    My questions are: - Will a proposed bracket be able to support 7500 lb. without undue or permanent distortion or failure? - Can the proposed dimensions be improved? The bracket will be attached to a large tree and will support one end of the 4x6 redwood swingset beam. (On the other end...
  14. J

    Steel Mounting Bracket Question

    I need a hand choosing the right size, thickness and shape of steel for a bracket to mount an old style 5 gallon Jerry water can on a Jeep. I estimate the weight of the can + water + misc = 50 to 55 pounds. The bracket will be subjected to rough off-road conditions. In the drawing the red...
  15. V

    Calc Poisson Bracket: {π,∂φ} Calculation

    How can I work out {π,∂φ} where {,} is a Poisson Bracket; π is the canonical momentum and ∂φ is the spatial derivative of the field (ie. not including the temporal one). Basically the question boils down to (or atleast I think it does!), working out ∂(∂φ) /∂φ - ie. differentiating the...
  16. I

    Verifying Lie Bracket for Vector Fields on U

    If we have vect (u) which denotes an infinite-dimensional vector space of all vector fields on u. As infinitesimal elements of the continuous group of Diff(u) they form a Lie Algebra. We then can define the bracket of two vector fields in v and w. If in coordinates: v = \sum_{i}V i...
  17. M

    Solving the Lie Bracket Question in Quantum Mechanics

    Hi! I was doing an assignment in quantum mechanics and came upon the following fact I cannot explain to me. I hope someone of you can and will be willing to :) Consider the creation and annihilation operators: a^+ and a and also the momentum and position operators p and x...
  18. P

    Maximum Force for Block on Bracket Friction?

    Homework Statement A 10kg block is resting on a 5kg bracket, which rests on a frictionless surface. The coef. of static and kinetic friction between the block and bracket are .4 and .3, respectively. Find out a) the max force that can be applied to the block without the block sliding on the...
  19. C

    Loci for z - curly bracket use

    Hello, I'm just unsure about the use of curly brackets in expressing loci of complex number z, to which I've been introduced in our calculus class: - I can't find elaborations on this on-line :confused: - No material I can find in our textbooks(?) I do know it is used to express the...
  20. I

    Proof about commutator bracket

    i've never really done a proof by induction but i would like to prove a statement about commutator relations so can you please check my proof: claim: [A,B^n]=nB^{n-1}[A,B] if [A,B]=k\cdot I where A,B are operators, I is the identity and k is any scalar. proof: [A,B^2] = [A,B]B+B[A,B] =...
  21. C

    Calculate the rotation "theta" at O

    Homework Statement a frame is composed as it is shown in the figure, each of length L. bending stiffness of AO is EI, of OB is 3EI. Force P is acting in the middle of AO, i.e at L/2. the question is to calculate the rotation "theta" at O. Homework Equations The Attempt at a...
  22. 7

    Brake caliper bracket mount - single shear bolt question

    I'm installing a set of front brake calipers on my track car, and the mounting brackets are made of aluminum (7075). What is the better mounting scenario - Thread the mounting bolts directly into the aluminum, or use a through hole mounting method with a proper bolt and nut combination? The...
  23. DaveC426913

    Mounting big-screen TV on wall bracket

    I have a Samsung 42" similar to this one: http://www.hotchickshotpicks.com/wp-content/uploads/2007/09/samsung-42-inch-tv.jpg except that of course mine doesn't show sports. I've mounted it on a wall support but the TV does not seem stable. I can grab it and wobble it. The wall support is...
  24. D

    What is the maximum load capacity for my fabricated bracket?

    Hi Guys I hope you can help. I’m currently trying to modify some existing equipment and have a problem that I hope you good people can help me solve. I have a fabricated bracket that currently supports another bracket than runs on linear bearings to position its self. see attached JPG for...
  25. C

    Poission Bracket of Angular Momentum

    Homework Statement Parts vii and viii of problem 2 of the attached file "hw5.pdf". It's too long and I'm too lazy to type it here.Homework Equations A useful equality of Levi-Civita symbol: \epsilon_{ijk}\epsilon_{i'j'k}=\delta_{ii'}\delta_{jj'} - \delta_{ij'}\delta_{ji'}The Attempt at a...
  26. M

    What bracket is used to denote a number is excluded from a domain?

    When asked to find the domain, what bracket is used to denote that a number is excluded or included in the domain, ( or [?
  27. D

    Poisson bracket significance (Classical Mechanics)

    We have to show that [Lx,Ly] = Lz [Ly,Lx] = -Lz [Lx,Lx] = 0 and I have done this. We then need to comment on the significance of these results, which I'm not sure of. I know in QM you get similar results for commutators of these quantities, and it means that you can't simultaneously know...
  28. K

    Defining GR with Poisson Bracket

    If you have the metric g_{ab} , \pi _{ab} as the metric and "generalized momenta", my question is if you can define GR using Poisson Bracet: \dot g_{ab} =[g_{ab},H] \dot \pi _{ab}=[\pi _{ab},H] and hence use these equations to obtain and solve the metric.:shy:
  29. J

    Solving Horizontal Force of Screw on Shelf Bracket

    A shelf bracket is mounted on a vertical wall by a single screw, as shown in Figure P12.59. Neglecting the weight of the bracket, find the horizontal component of the force that the screw exerts on the bracket when an F = 86.0 N vertical force is applied as shown. Ok so we have three...
  30. A

    Solving Positive Integer Problems: First & Last Terms in nth Bracket

    Hi everyone, This is my first post here :smile: Anyway I have problems solving this question wonder anyone could help give me some clues as to how to go about it. Here goes: The positive integers are bracketed as follows, (1), (2,3), (4,5,6,7), (8,9,10,11,12,13,14,15), ...
  31. S

    About the basics of Poisson bracket

    Dear all, Please help me to solve the following problems about Poisson brackets. Let M be a 2n-manifold and w is a closed non-degenerate di®eren- tial 2-form. (Locally we write w = w_ij dx^i ^ dx^j with [w_ij ] being a non-degenerate anti-symmetric real matrix-valued local function on M)...
  32. L

    Non-degenerate Poisson bracket and even-dimensional manifold

    From this reference: titled From Classical to Quantum Mechanics, I quote the following: ( \xi^i are coordinate functions) Let M be a manifold of dimension n. If we consider a non-degenerate Poisson bracket, i.e. such that \{\xi^i,\xi^j\} \equiv \omega^i^j is an inversible...
  33. lethe

    Pushforward of Lie bracket

    one elementary result that you see when you first learn differential geometry is that the pushforward of the Lie bracket of two vector fields is the Lie bracket of the pushforward of the two vector fields, i.e. let \phi be a diffeomorphism from manifold M to N, and let v, w be two vector...
Back
Top